Search found 250 matches


Vishal please check the attachment.I have tried to explain but dont know how much i am correct.
Hope it helps you.

by saurabhmahajan

Fri Nov 19, 2010 9:48 am
Forum: Problem Solving
Topic: GMATPREP - PS problem -3
Replies: 4
Views: 1233

GMATPrep3

Please help with this..

by saurabhmahajan

Tue Nov 16, 2010 1:48 am
Forum: Problem Solving
Topic: GMATPrep3
Replies: 1
Views: 1126

GMATPrep 2

Please help with this..

by saurabhmahajan

Tue Nov 16, 2010 1:48 am
Forum: Problem Solving
Topic: GMATPrep 2
Replies: 1
Views: 986
by saurabhmahajan

Tue Nov 16, 2010 1:47 am
Forum: Problem Solving
Topic: GMAT PREP problem
Replies: 3
Views: 1367

SC problem....BIG PROBLEM

Friends, I am facing problem in solving SC questions in GMAT Prep software. My accuracy level when solving SC questions on this forum or from OG has been good, say 8 out 10 i get correct. But when i take GMAT Prep test it decreses to as low as 5 out 10. So just out of curiosity, is there diff betwee...

by saurabhmahajan

Mon Nov 15, 2010 3:36 am
Forum: GMAT Strategy
Topic: SC problem....BIG PROBLEM
Replies: 3
Views: 1231

well you can do it by plugging values from options..... First take the middle value, say oil a is 290 ounces. So 5% of additives will be 14.5 (290*.05). to make it 30 ounces of additive we now require more 15.5 ounces of additives from oil B. so the remaining 190 (480-290) should give us 15.5. 190*....

by saurabhmahajan

Thu Nov 11, 2010 4:08 am
Forum: Problem Solving
Topic: A certain industrial machine requires 15 quarts (Grockit)
Replies: 3
Views: 969

my way:

8 kg solution Y. in this 30% X so X = 2.4 kg
afetr removing 2 kg water again 2 kg added.
Again 2 kg of solution Y added, so 30% X now becomes = 0.6 kg
so total X= 2.4+0.6 = 3
now 3/8 = 37.5 %

by saurabhmahajan

Mon Nov 08, 2010 11:24 pm
Forum: Problem Solving
Topic: Hard Solutions/Mixture Problem
Replies: 24
Views: 20027

I too got 42.
Let the lion be L1 & L2
for L1 total 8 choices
for L2 7 choices
So total choices 56 (8*7)
But they can sit adjacent so combining them will leave only 7 choices and as they can be L1-L2 or L2-L1 so total combined choices will be 14.
Now 56-14 = 42

by saurabhmahajan

Mon Nov 08, 2010 10:38 pm
Forum: Problem Solving
Topic: Combination Problem
Replies: 5
Views: 1469

Vikram A is wrong becoz politician P does not advocates or present any of his own views.He just counter attacks his opponent's view. and thus he is not defending his view. which A says so.

by saurabhmahajan

Wed Oct 27, 2010 2:21 am
Forum: Critical Reasoning
Topic: Politician P.
Replies: 5
Views: 1841

I too went for B.... :-(

are you sure the OA is D ?

If Yes then we need CBI to find out why D.

by saurabhmahajan

Wed Oct 27, 2010 1:50 am
Forum: Critical Reasoning
Topic: an CR from Gmatclub
Replies: 2
Views: 1365

IMO: C

becoz Politician P mentions that people would be upset and concludes that the opponent is mistaken.

OPTION C proves this

OA please.

by saurabhmahajan

Wed Oct 27, 2010 1:37 am
Forum: Critical Reasoning
Topic: Politician P.
Replies: 5
Views: 1841

for Question No. 15, Test A, 1000 CR

Alba first makes comments agianst senator frank for soft on crime. but after tam makes a point in support for senator frank, alba refuses to accept it by giving unsupport statement...
Thus option C is my choice also.

by saurabhmahajan

Tue Oct 26, 2010 3:48 am
Forum: Critical Reasoning
Topic: 1000 CR_01 Explanations would be appreciated
Replies: 3
Views: 1335

for Question No. 14, Test A, 1000 CR

To correct this, the United States must keep its resource consumption at present levels
this sentence directly makes a point that US uses most of the resources. And this is supplemented by option B

by saurabhmahajan

Tue Oct 26, 2010 3:35 am
Forum: Critical Reasoning
Topic: 1000 CR_01 Explanations would be appreciated
Replies: 3
Views: 1335

for Question No. 8, Test A, 1000 CR As you know, funding for summer jobs is limited, and it is impossible for us to offer jobs to all those who want them. Consequently, we are forced to reject many highly qualified applicants. this statement clearly gives us a clue that there were more applications ...

by saurabhmahajan

Tue Oct 26, 2010 3:29 am
Forum: Critical Reasoning
Topic: 1000 CR_01 Explanations would be appreciated
Replies: 3
Views: 1335